Thursday, September 3, 2020

Garth Brooks essays

Garth Brooks papers Troyal Garth Brooks was conceived in Tulsa, Oklahoma of February 7, 1962. He was brought up in Yukon around one hundred miles from Tulsa. Beam, his dad, functioned as a sketcher for an oil organization while his mom Colleen was a nation vocalist during the 1950s and showed up on radio and network show, just as recording different accounts for Capital Records. Down home music assumed a job in the Brooks' family unit, yet not a prevailing one. However, when Garth was conceived, his mom had resigned from an expert vocation and the Brooks' home resonated with as much stone and popular music as nation. Garth went to Yukon High School and played four games: football, b-ball, baseball and track. He headed off to college at Oklahoma State in Stillwater and no set designs for a music vocation. It was in 1984, when Garth understood that his games profession was not going anyplace, so he studied promoting, took some advertising classes, and began to pay attention to music. He started to act in bars and honky-tonks around Stillwater. In the mid year of 1985, he left for Nashville and a vocation in blue grass music, however got back four days after the fact dismissed. After, he joined a nearby band Santa Fe as the lead artist. During this time Garth turned out to be enormous enthusiasts of artist journalists James Taylor, Dan Fogelberg, musical crews Boston and Kansas, and nation artist George Strait. In 1986, Garth wedded Sandy Mahl. They met when Garth functioned as a bouncer at a bar. At that point in 1987, Garth, with his new spouse, and Santa Fe, moved to Nashville. Subsequent to recording numerous demos for distributers, Garth marked an essayist's agreement in November of 1987 and not long after met Bob Doyle, who later turned into his administrator. It was Doyle who paid the passage charge to a Bluebird Cafe. The exhibition at this Cafe earned Garth his first record manage Capital Records. He at that point discharged his collection Garth Brooks in 1989. This collection gave no sign of what was available for Garth not far off. During th ... <!

Wednesday, August 26, 2020

The US Financial Crisis A personal Revelation Free Essays

After the Second World War, nobody ever however that the dad of majority rules system and the most evolved country on the planet could ever be influenced by an emergency emerging from monetary issues.â The development of US and its situation as a force to be reckoned with was because of the focal job it played in the universal war and the advancements it made from that point. Dreaded by its adversaries and regarded by its partners, the US was and still is the jealousy everything being equal. We will compose a custom article test on The US Financial Crisis: An individual Revelation or then again any comparative point just for you Request Now  Though still the most evolved country and a money related influence, the 2008 emergency which was a consequence of a mix of variables including a fall of sub prime, disappointment of monetary foundations, high oil costs, agonizing paces of swelling and the disappointment of significant ventures. The request introduced is very like the genuine occasions however there were impressive overlaps.â The US economy is severely influenced, numerous employments have been lost and to manage the impacts o the emergency the US should create hearty monetary systems. Any ordinary society can be isolated into two significant classifications: poor people and the rich.â This is a categorisation whose premise is asset capacity and impact is a genuine portrayal of the truth on the ground.â Happiness is thought to be legitimately identified with money related capacity and all that a great many people look for in their lives is asset strengthening.  A gander at the US shows that the reason for disappointment of its framework emerged from a solitary reason which multiplied.â If one should fall into the allurement that the course to bliss is storing up riches at all cost, at that point they are similarly as simple to disappointment for such a methodology was the purpose for disappointment of sub prime home loans. Indeed, even the wealthy in the general public have their own difficult which may even be more noteworthy than those of poor people; an underdeveloped country is less inclined to be looked by an emergency of such size. At the point when one is building up his wealth, he ought to guarantee that he creates other emotionally supportive networks for in all actuality the mission for cash has no reasonable connection to happiness.â Happiness is a perspective and one can't legitimize that by being rich he will carry on with his life an upbeat individual for as at now the two Americans and Iraqis are languishing. The US case has been an individual disclosure on the idea of life and what does entail.â Life is mind boggling and achievement doesn't promise one will live a difficulty free life.â The strength that the Americans have appeared even with misfortune ought to be embraced by all individuals venturing through life.â The information that all are defenseless to challenges is significant in creating trust which is focal in to the whole recuperation process. The US way to deal with the emergency is a gem to behold.â The US recognized it had an emergency and took important estimates that didn't really include the assistance of other country rather the components executed inside created frameworks to manage the emergency. Consider it the individual will to manage an issue or being able to; it is possible that one ought to never disregard their issues yet ought to recognize that a difficult exist and take pertinent measure to manage the issue. Shouting for help isn't generally the best way to deal with managing individual issues for we as a whole have unbounded individual capacity that we should create in the midst of bounty and emplo Step by step instructions to refer to The US Financial Crisis: An individual Revelation, Papers

Saturday, August 22, 2020

Evolution Lab free essay sample

Did the development of the phylogenetic tree dependent on the 14 living Caminalcules change the manner in which you would bunch Caminalcules species in your ordered arrangement (diagram)? What does this propose about arrangement dependent on carefully of similitude versus transformative relationship? The phylogenetic tree was a greater amount of a developmental tree. It indicated the development of the life forms and their progenitors or qualities that they expand from. The graph anyway basically arranges and isolates the life forms in comparability. The tree didn't change the outline, the graph see likenesses with structures between the cutting edge creatures and the tree shows their transformative linage. The tree mirrored our ordered characterization. 2. Did your phylogenetic tree dependent on the 14 living Caminalcules contrast from the phylogenetic tree based all Caminalcules, both living and fossil? Which do you have more trust in? What do fossils add to our comprehension of Caminalcules development as far as rates and bearing of transformative change? The two trees were genuinely extraordinary. We will compose a custom article test on Development Lab or then again any comparable subject explicitly for you Don't WasteYour Time Recruit WRITER Just 13.90/page That is because of the more measure of data that was given by means of the fossils. The tree with all the caminialcules unmistakably indicated when a trademark vanishes and a trademark proceeds onward. We are increasingly sure about the tree with both living and fossil since it permitted us to see where the qualities of the living beings vanished or proceeded onward. The fossils gave us that when a key attribute remains in a life form it lets us to anticipate the heading of the transformative change when taking a gander at advanced living beings. 3. Distinguish the latest progenitor of any two species. (e. g. pecies 5 is the latest predecessor of species 2 and species 4) species 75 is the latest progenitor of species 4 and 3 4. You will see that a few heredities stretched commonly and are spoken to by many living species. Give a model from your phylogenic tree of this quick expansion. Species 43 fan out to 8 distinctive cutting edge species. 5. A few genealogies (e. g. the relatives of species 58) changed next to no after some time. A genuine case of this would be â€Å"living fossils† like the horseshoe crab or cockroach. Give a model from your phylogenic tree of developmental balance. Talk about the environmental conditions that may bring about this kind of long haul transformative balance. A case of transformative dependability would be species 13, the species endure in excess of 13 million years and now is a â€Å"living fossil†. The capacity for the species to duplicate and get by in a territory implies a there is no requirement for the species to advance. Constant natural conditions brings about this drawn out balance. 6. Some Caminalcules went terminated without leaving descendents. In reality, what normal for an animal categories (not of their enivornment) may build the likelihood of an animal categories going wiped out? The environmental conditions may have changed rapidly and the species couldn't adjust to those conditions, which in the long run lead the species to go wiped out. 7. Discover two instances of concurrent development among the Caminalcules. This implies discovering situations where at least two species have a comparative trademark that developed autonomously in every heredity. The wings of bats, feathered creatures and honey bees is a case of assembly since the three gatherings didn't acquire the trademark from their normal predecessor. Compose your answers in complete sentences (e. g. Species x and y both have  but their latest basic predecessor, z, did not†). The paws of species 3 and 12 (their latest regular progenitor, species 46, didn't have hooks) The wings of 61 and 51 and of 19 and 20. 8. Depict two instances of minimal structures that you can discover among the Caminalcules. These are structures that have been diminished to the point that they are essentially futile. E ar muscles and the tail bones are instances of minimal structures in people. Clarify how minimal structures give pieces of information about an animal categories? transformative past. Delineate your contention with minimal structures found in people or other genuine species. The decreased tail of species 3 and the diminished number of digit or finger of species 35 and 66. Minimal structures give a great deal of pieces of information, since it is a characteristic that has been carried on for ages, it very well may be connected back to their predecessors when taking a gander at fossils. A human minimal structure can be the sixth finger, which is a predominant characteristic, however because of regular choice it is seldom observed. The male areola is another case of a practically pointless organ or structure.

Research Proposal on Drug Trafficking Essay Example

Research Proposal on Drug Trafficking Essay Medication dealing is the illicit creation, transportation, and exchange with drugs. It is obvious that the presence of medication dealing is the worldwide issue which requires quick arrangements. The unsafe effect of medications is known everywhere throughout the world, that is the reason most of medications, even the lightest of them are restricted. Medications are known to cause significant mental impact of the individual, and the vast majority of them produce habit so the individual goes through her cash to buy an ever increasing number of medications since she can not get by without them on the mental and physical level. Medication dealing is likely the most beneficial business since billions of dollars are spent on them everywhere throughout the world yearly. The salary got from tranquilize dealing will never improve the financial aspects of the nation since this business is frequently constrained by serious lawbreakers who utilize the money for their great. Medication dealing is a risky marvel which can ruin the monetary improvement of the nation in such a case that a noteworthy number of dealers are packed in a specific spot, the creation rates lessen due to the expansion of the name of the medication addicts. No big surprise, the nations of South America, Africa, South Italy have monetary issues being the focuses of the world medication dealing. Medication dealing is a risky wonder not in light of the unsafe impact of medications on the human wellbeing, but since of the various social issues which are brought about by drugs: enslavement, wrongdoing, joblessness, suicides, prostitution, and so on. The medication junkie is prepared to do everything to buy the accompanying portion of medications and begins to carry out wrongdoings if there are no other options. We will compose a custom paper test on Research Proposal on Drug Trafficking explicitly for you for just $16.38 $13.9/page Request now We will compose a custom paper test on Research Proposal on Drug Trafficking explicitly for you FOR ONLY $16.38 $13.9/page Recruit Writer We will compose a custom paper test on Research Proposal on Drug Trafficking explicitly for you FOR ONLY $16.38 $13.9/page Recruit Writer Medication dealing is a difficult issue which requires an answer. The understudy can attempt to unravel the response to this everlasting inquiry finishing an examination proposition which would recommend the investigation of the most significant and irksome issues of the issue. One should contemplate the history, geology and social parts of medication dealing, assess the outcomes of this wonder, watch the techniques and endeavors which are taken to lessen the effect of this issue. The quality and the correct strategies ought to be applied during the time spent the examination to convince the teacher of the direness of the subject. The youthful expert can prevail during the time spent composing depending on the top notch very much organized free model research proposition on tranquilize dealing formed by the genuine master in this issue. It is a savvy choice to exploit a free example inquire about proposition on tranquilize dealing which can show the understudy to make the content expertly and configuration the paper as indicated by the measures of his instructive organization. In the event that you need a PhD degree master to assist you with scholastic proposition composing, check the accompanying composing administration:

Friday, August 21, 2020

Art Imitating Life Imitating A essays

Craftsmanship Imitating Life Imitating A papers The late 1950s saw another development in the workmanship world this got known as pop craftsmanship because of the way that the specialists in this development with this development masculine Andy Warhol and Roy Lichensten of the joins states just as David Hockney and Derek Boshier of Great Britain, utilized components of mainstream society as principle wellsprings of their work. A genuine case of this is Warhole's screen prints of Marilyn Manroe, where he took a well known symbol of the time and utilized a large scale manufacturing method to make her into a masterpiece. Lichenstine took a gander at an alternate component of mainstream society youth culture-comic books. He utilized this style of painting and attracting to make a funny cartoon picture. For instance Wham, in spite of the fact that looks screen-printed it was infact gasped by hand speck by spot. Hockney was impacted by spray painting a section ignored by his counterparts, he utilized it as messages scribbled over the foundation of his artistic creations . Hockney said that he did this so as to make it understood to the watcher what the image was about. Derek Boshier painted a progression of pieces dependent on thoughts he got from a toothpaste business. identi-pack man was his method of proposing how simple it was for organizations to control individuals with the utilization of publicizing and make mass markets. The entire thoughts of pop workmanship, as to take things from regular daily existence and make them into a bit of fine art. These things were those, which the individuals of post-war Britain were not prepared to think about craftsmanship at all. There was a thing like Campbell's soup container of Warhol, The kitchen utensils of Tom Wesselman and Richard Hamilton and kid like scrawls of David Hockney. Because of the craftsmanship blast, during the 1960s and clear patterns through pop workmanship That originators had the option to get on these and consolidate them into their work, impacted by design symbols, famous actors, innovation, and motorcars. In spite of the fact that the craftsmen had some covering styles, pop craftsmanship concentrates more regarding the matter and less on styl... <!

Saturday, August 15, 2020

Thoughts from the Other Side

Thoughts from the Other Side I know Ive practically disappeared off the blogs BUT I promise I have had good reasons! As some of you may know, Ive been studying abroad at the other Cambridge, Cambridge University, for the past almost 6 months. Here at Cambridge, instead of having finals at the end of each semester, like we do at MIT, they have ALL their finals from classes they took in their fall term (Michaelmas term) and spring term (Lent term) at the end of the year (Easter term). Since each student generally takes 5 classes a term, most third year engineering students have 10 exams. ALSO, these 1.5 hour exams are pretty much  100% of your grade in the class. Yeah But I was lucky and only had 5 exams since I came in during the middle of the year. Of course, I say only 5 and thats more than Ive ever taken at MIT. Anyways, one of the attractive portions of the Cambridge-MIT Exchange is that we get this 5 week long break between Lent Term and Easter Term but Third year engineers have their exams at the beginning of the term which means we spend a lot of time over break reviewing our notes and studying. I wasnt as crazy as some of the people here who would stay in the library until midnight and be back in the library at 7 am but it was a pretty boring month of my life. We tried to keep ourselves sane by exercising a lot and reminding ourselves that this was going to be us after our exams were finished After all, we only had four weeks of projects. How bad could those be? So we just forced ourselves to buckle down and do the work When I finally walked out of my last final, this was me: I was so happy. It was an amazing feeling. One of the traditions here at Cambridge is that people spray their friends with champagne after they get out of their last exam. Ernesto R. 15, whos on the exhange with me, got these great photos of my friends getting sprayed For two glorious days, us engineers relaxed and enjoyed the sunshine. It was magical. Then projects started and only then did I realize In true Cambridge fashion, projects have turned out to be A TON OF WORK (ie, Im working on the weekends) which can be described in no other manner than just sad. The amount of people I see in the department computer lab during odd hours is sad. My eyes are sad from staring at a computer screen for such long hours. So. much. sad. I feel like a tiny bit of my soul dies every time I walk into lab, but that might just be because Im feeling my days here are numbered. That all being said, I have been making time to do some awesome things at night :) Ive baked a ton of really delicious desserts with my friends and explored the city. (Ernesto R. 15  and Teresa L. 15 have pretty much been my baking buddies.) Thats one of the great things that Ive really enjoyed about Cambridge- Being able to explore such a beautiful little city on bike and having time to bake tons of delcious things. I do like how I have more time for myself here- which I think is really important to learn more about myself. So thats why I havent been blogging too much.  But those of you reading the blogs are probably here expecting to read about MIT and not Cambridge University, so Ive compiled some interesting revelations about MIT during my time here that I thought Id share. 1. MIT wants to help you succeed. Part of this comes down to the fact that Ive adjusted to learning in the MIT system, but I do feel like there are more ways at MIT where you can get help if you want to. Office hours (an hour or two where professors or teaching assistants are available to help students with questions if they have any) have literally saved me so many times during my time at MIT. When Im just stuck beyond all belief on a problem, it was easy to get help if I needed it. We dont have office hours at Cambridge, and I found that I really missed them. At Cambridge, they have these things called supervisions, where you and another student meet with a graduate student to review your assigned work. While I did enjoy the smaller group sizes I never felt they were as helpful as office hours because you couldnt get help while you were working on your assigned work. I did have FANTASTIC supervisors though, and they did a good job of making sure I understood the problems I did incorrectly during the hour review. It was also nice to be able to ask all the questions you did have about your work, because sometimes in office hours there are too many people to be able to ask all your questions. On another note, I felt like there was more of an emphasis on mental health at MIT. People talk about it more and are more aware of the stresses of a difficult academic environment. Plus, we have resources where people who are struggling can go get help. That being said, at Cambridge, youre assigned a director of studies at your college who you can talk to about these kinds of things. It is nice knowing you have one person who knows you a little better and might be easier to talk to about personal matters. 2. Your peers want to help you succeed. MIT students definitely collaborate a lot more than here at Cambridge. We have these bi-weekly problem sets (which are called examples papers here), and I found that most people dont really work together on them. Maybe its beacuse examples papers not graded and only for your own learning, but I dont think many people really worked on things together in the same way they did at MIT. People at MIT are more willing to admit that theyre struggling with something and work with you on the tough problems. Ive missed that collaborative work environment here. Additionally, Ive found Cambridge students are more competitive when it comes to grades (they refer to them as marks), but not overtly so. Here, your marks are pretty much public, so everyone knows how youre doing. Its a little weird, because sometimes it feels like it makes people less willing to help each other get better marks. At MIT,  people generally dont know other students grades (unless youve explicitly asked them) which is nic e. That all being said, the Cambridge engineers Ive met here are awesome people, and I wouldnt have been able to transition to the new learning environment as quickly without them! 3. The work is long and hard, but thats how you learn. As weird as it sounds, I think I learn best when I had those darn weekly problem sets. The constant feed of work actually helped me keep more accountable about learning the material and gave me a LOT of experience working on problems (which I just felt I lacked here). Nothing was worse than getting to the beginning of time to study for exams here and actually feeling like I could do very little (but that might have been because I wasnt doing enough revision of my notes during the term). Ive never felt that way about MIT final exams, since we usually have only a day or two before they begin, and I think I have the weekly problem sets to thank for that. Although I used to complain about the lack of time to study for exams, the truth is I probably never never really needed that much time. I think it really comes down to the fact that Im not used to the self studying style here at Cambridge. I think a lot of the students here are used to it, which is amazing! I wish I could sit down and study with the same focus that they have. 4. MIT has incredible resources that they let you use The amount of software and equipment available to MIT students is amazing. After spending time here at Cambridge, Im so thankful about at all the software we can get through MIT (like Microsoft Office, Solidworks, etc). Im also so thankful for all the neat projects, experiments, and research that Ive gotten to do through MIT. For example last semester, I took 2.008 (a Design and Manufacturing class) in which I got to use an injection molding machine to make yo-yos. Thats something that people dont do get to do here. A larger majority of the work is done on the computers. I miss my hands-on work! All that being said, I really have loved my experience here at Cambridge University. I like how to Cambridge culture doesnt revolve around work. People work hard, but also spend a lot more time doing things they enjoy outside of work whether that be running, biking, drawing, baking, cooking, reading, etc. Its nice to be in an environment where people can separate work and life a little better. PLUS people actually hang out during the week! Crazy. Ive also met some incredible people and participated in some Cambridge traditions that Im going to miss when I go back. One of the Cambridge traditions Ill definitely miss will be formals, which are these three course dinners that each college hosts during the week. My friends and I went to Jesus College formal a couple days ago. Here we all are in our snazzy gowns. Andy, Teresa L. 15, Sarah M. 15, a blurry Ernesto R. 15, Nati, and I at Jesus formal. Im not ready to leave yet! The good news is that after next Saturday, I will be done with projects and everyone wil be done with their exams! Normally at the end of finals, all the MIT students are pretty much like.. . so everyone wants to leave MIT as soon as possible, arrive at home, then sleep for a million hours. But here at Cambridge, everyone celebrates with Garden Parties and these massive celebrations called May Balls (which are actually in June. Silly Cambridge). I cant wait!

Sunday, June 21, 2020

Cognitive and Psychosocial Development During Adolescence

Cognitive and Psychosocial Development During Adolescence Cognitive Development of Adolescents Adolescence is considered as a stage wherein a person transitions into an adult (Green Peal, 2010). At this stage, the person experiences significant changes not only in the physical aspect, but also in the cognitive aspect. As the study about adolescent cognitive development have been conducted by the Swiss psychologist, Jean Piaget. In his theory of cognitive development, Piaget proposed that the mental capability of children develops with age. According to Piaget, there are four major stages of cognitive development wherein children can be classified into. Children between the ages 0 to 2 years old are classified under the sensorimotor stage. At this stage, learning occurs through a trial and error process (Grossniklaus, Smith, Wood, 2011). The second stage of Piaget’s cognitive development is what he calls as the preoperational staged. Children who are between the ages 2 years old and 7 years old are included in the preoperational classification and are believed to have already developed the capability to communicate, memorize and imagine (Grossniklaus et al., 2011). Children at the preoperational stage are believed to possess egocentric behaviors and are beginning to develop their logical intelligence. Children between 7 to 11 years old are believed to be already self-aware. They now have the ability to reason and determine the consequences of their actions. Piaget classifies this stage of cognitive development a s concrete operational. The last stage of cognitive development, according to Piaget, is the formal operational stage (Green Peal, 2010). Under this stage, children are believed to possess the ability to understand and formulate abstract ideas, such as morality, justice and conscience. Piaget’s formal operational stage is critical because it corresponds to the adolescent stage, which encompass children between 11 and 15 years old. It is the stage wherein most children reach the peak of their thought processes. Children, at this stage already have the ability to hypothesize. As observed by one scholar, formal operational thinkers are able to â€Å"construct hypothesis to account for a particular phenomena, deduce from these hypothesis that certain events should occur, and test the hypothesis by finding out if the events do occur† (Day, 1981, p. 45). Formal thinkers also begin to understand abstract ideas, such as love, faith, morality and spirituality (Grossniklaus et al., 2011). Some scholars associate the cognitive development of adolescence to the physical changes that occurs in the brain (Steinberg, 2005, p. 70). Because of this heightened ability to think for themselves, many adolescents become self-conscious. They tend to think that all eyes are on them and that they are the focus of other people’s attention, which accounts for awkwardness, especially in public. Adolescents also tend to think that their experiences are unique and that no one understands them. Psychosocial Development in Adolescence An adolescent’s psychological and social development is influenced by many converging factors, which includes their cognitive and emotional experiences. These experiences shape their behavior towards their family, peers and in public. According to Stanley Hall, a renowned American psychologist, individuals under the adolescent stage experience psychological and emotional turmoil, which he refers as ‘storm and stress’ (Arnett, 1999, p. 317). For Hall, this psychological and emotional turbulence accounts for the observed behavioral problems, such as mood swings, irrationality and impulsiveness. Scholars have also observed five major concerns that shape the psychosocial development of adolescents. Accordingly, adolescents are concerned about establishing their own identity; establishing their autonomy; establishing intimacy; concerns about their own sexuality; and concerns about their achievements (Ruffin, 2009, p. 4). It is quite common, for instance, for adolescen ts to experience confusion regarding their personality, primarily because of the many changes that occurs in their physical, mental and emotional state. Some adolescents, even exhibit delinquent behaviors; engaging in risk taking activities. Others, on the other hand, could not get along well with their parents; possessing a mindset that their parents could not understand their emotions. As a result, most adolescents spend more time with their friends as compared to the time they spend at home with their parents. Some get involved in drugs and substance abuse along with their peers. For Hall, such anti-social behaviors are associated with the emotional stress that occurs during adolescence, which eventually subsides as the individual matures (Arnett, 1999, p. 317). The turbulent psychosocial stage that adolescents go through should not be ignored. It is during the adolescence stage, for instance, when most individuals start to get involved with gangs, crime and drugs. Others struggle with anxiety and depression to the point that they would attempt to end their lives. The risk of suicide among adolescents is high. Studies in the United States, for instance, have observed that suicide is the leading cause of death for most young people between the ages 15 to 25 years old (Blum Qureshi, 2011). More than ever, adolescence is the stage wherein the child needs the emotional and moral support of their parents. Consistent with the social disorganization theory, the breakdown of social institutions, particularly the family, could impact the psychological and social development of children (Kubrin, 2009, p. 226). Failed parenting, divorce and dysfunctional families, for instance, are seen as the common grounds of delinquency among adolescents. Support g roups and social institutions, such as the school and the community, can also provide a nurturing effect on the development of individuals in this tumultuous period. The breakdown or failure of these institutions to provide the necessary intervention could also influence the psychosocial development of adolescents.

Sunday, May 24, 2020

Dementia Patients Denied Rights - 1622 Words

Dementia Patients’ Denied Rights: Voluntarily Stopping Eating And Drinking Adrienne Heasty County College of Morris The healthcare system places emphasis on involving patients in their plan care throughout the disease or healing process. Nurses and other members of the healthcare team are responsible to ensure that the competent patient has the right to refuse any medical treatment. Patients can have an active voice in their treatment throughout their disease process by clearly stating their treatment requests in an advanced directive. Patients who file advanced directives are warranting their current wishes are met in the event that they are no longer able to make decisions for themselves. However, an advanced directive (AD)†¦show more content†¦Although Sheri carefully articulated her wishes for end of life care, she ended up in a circumstance all too common for people with dementia. There is a limbo for dementia patients; where their decision making for end of life care â€Å"typically lies between the last opportunity to act decisively and the time when dementia’s severity is deemed to make death preferable.† (Menzel and Chandler-Cramer 2014, p.25) The nursing home staff argues that Sheri’s â€Å"then-self† may have had different wishes than her â€Å"now-self† and there is no clear way for Sheri to currently communicate her current demands. Therefore, the death Sheri requested was denied and she will live out the full course of her dementia in a nursing home. Sheri is not alone in her fight to hasten death. There are many stories similar to hers. One in particular is the case of another person with dementia named Judge Robert Hammerman. (Pope 2011) Hammerman defined a life of living with dementia as breathing, not really living. (Pope 2011, p. 374) He described his battle with dementia and highlighted the limitations he faced daily. Losing his memory, having a harder time completing simple tasks, and the possibly of needing to be institutionalized all contributed to his decision of taking his own life, without medical assist ance. He did not have the same end of life options as other illnesses, such as cancer, and committed suicide. Perhaps if Hammerman was able to ensureShow MoreRelatedThe Effects Of Artificial Nutrition On Health Professionals981 Words   |  4 PagesImagine being admitted to the hospital and being denied your right to have food and water. While it is known that adequate fluid and nutrition is needed for survival, many health professionals are currently going through an ethical dilemma of whether or not to provide artificial tube feeding for patients with terminal diseases. This ethical decision that affects health professionals must take into consideration the recent research that opposes ANH for this population. These studies have demonstratedRead MoreCase 5 History and Physical Essay1632 Words   |  7 PagesHISTORY AND PHYSICAL EXAMINATION Patient Name: Fanny Copeland Patient ID: 115463 DOB: 10/26/- - - - Age: 58 Sex: Female Room No.: Hillcrest Memory Diagnostic Center Date of Admission/Date of Arrival: 04/26/- - - - Referring Physician: Lyndon F. Talcott, MD, Neurology Admitting Diagnosis: Memory loss. BACKGROUND: Ms. Copeland is a 58-year-old left handed white female who was referred to the Hillcrest memory Diagnostic Center by the emergency room physician for evaluationRead MoreHow Alzheimer Is Like Mental Cancer Essay1227 Words   |  5 Pagesanother question that is not speculation: When did we notice? When did it really start effecting his life? When did we get the first glimmers that something was wrong, even terribly wrong? The realization was progressive The signs were at first excused, denied, or minimized. Grandpa was getting old. Old people were forgetful. They made mistakes. I remember one incident that occurred years before anyone realized Grandpa had Alzheimer’s. It surely wasn’t the first sign, but it was one that lodged in my memoryRead MoreEuthanasia Persuasive Essay1064 Words   |  5 Pagesbecome disillusioned and hasty looking for a way out. Euthanasia offers a way out but, this pessimistic end could have been avoided by the constant advancement of medical science. It is a selfish, hopeless means of death that strips hope from other patients, fearing that euthanasia and death are their only options. Some may say that it preserves dignity but it is actually quite a selfish option that wounds families. On the film The Suicide Tourist, he pushes away his children away on his final momentsRead MoreEssay Euthanasia1563 Words   |  7 Pagesrecent years regarding euthanasia. It is a topic of great significance and sensitivity, because in the simplest terms, it is a debate about someone’s right to take his/her own life. Ultimately the legalization of euthanasia is a matter of human rights, and therefore the outcome of its debate has great implications on how humans define those inalienable rights. The arguments against euthanasia are numerous, and many of them are valid, good, humanitarian points. After all, euthanasia has been used to justifyRead MoreEssay on WHAT IS THE RIGHT-TO-DIE MOVEMENT?1300 Words   |  6 Pagesend-of-life or right-to-die and thus bringing complexity to an otherwise easy decision. But the most crucial question to ask is: are those in support of the right-to-die justified in their movement? This will be the question that will be addressed in this argumentative essay. Why oppose the right-to-die movement For those in opposition of the right-to-die movement (Ullmann-Margalit 73), there are a number of issues that they raise. It is often argued that most of those involved in right-to-die decisionsRead MorePotent Pharmaceuticals Of Terminal Dementia2409 Words   |  10 Pages Potent Pharmaceuticals in Terminal Dementia: The Benefits of Potent Opiates, Benzodiazepines, and Anti-psychotics in End-Stage Dementia Claire Bassett Excelsior College Abstract Potent pharmaceuticals like Haldol (haloperidol), Roxanol (morphine sulfate oral suspension), and Ativan (lorazepam) have clear benefits in Advanced Dementia (AD) patients. Though often contraindicated in elderly and demented patients, these and similar medications are frequently used in hospice and palliativeRead MoreThe Article What Broke My Father s Heart 1123 Words   |  5 Pagesdevice, but he denied her request. Eight years after his initial stroke, Jeffery passed due to pneumonia. But even after his passing, Jeffery’s pacemaker continued to work flawlessly. The issue is whether to respect the family’s autonomy and remove the pacemaker that is prolonging Jeffery’s life and create more suffering, or if this is beneficent. Today, pacemakers are a common treatment for cardiac rhythm problems and can be life sustaining. According to Zellner et al. (2009), â€Å"As patients age and sufferRead MoreTaking a Look at the Tuskegee Project1538 Words   |  6 Pagestest subjects. Testing was conducted by government doctors. The doctors advertised how these males would have the chance to be special government patients. The goal was to learn more about how syphilis attacks the body and how it evolves in black men over time and through multiple stages. The doctors performed physicals and blood tests on all patients. Doctors told the black men that they had â€Å"bad blood,† and promised to give these men free medicine and health care. The black men did not understandRead More Alzheimers Disease and Research: Ethical Concer ns Essay1904 Words   |  8 PagesCan patients with Alzheimer’s disease participate in clinical research? Theoretically, two radically opposite views regarding this issue can be posited. The first one, which is rather â€Å"conservative† could, in principle, argue that because of pervasive cognitive impairments AD patients are vulnerable and not capable of giving informed consent with a similar degree of responsibility as that of healthy individuals. When a surrogate’s decision is required for participation in research, this decision

Monday, May 18, 2020

Aristole - 962 Words

Ethics in Aristotle’s Philosophy Ruth Geter AC 504 Ethical Issues in Business and Accounting Unit 2 Assignment July 19, 2016 Aristotle Philosophy Introduction I have chosen Aristotle as my philosopher. â€Å"He was the first to argue that equals should be treated equally, and unequal’s should be treated unequally in proportion to their relevant differences† (Brooks and Dunn, 2014). In today’s society people sometimes lose sight of being ethical and often know what is right but chooses to not do the right thing. Most of the time it is out of greed or selfish reasons. His ethical theory is about being happy. â€Å"Of all the classical theories considered here, his is the farthest from an ethics of self-interest† (P.I.E,2006). Included in†¦show more content†¦I believe that Aristotle just wanted to see everyone happy in life and that’s why his theory and ethics were so important in treating everyone fairly and then everyone could be happy. Asking the tough questions help him and other’s he was asking the questions to question their ability to know the difference between right and wrong and if he or she was being ethical and doing what was best. I always believe we as society need to look at the bigger picture before making a decision and ask is this decision for the better good. Summary In summary, it is great to have an influential person such as Aristotle to read about his theories and ethical practices. Learning to ask questions to get the best result is very important. Even though our cultural backgrounds, or ethical or religious beliefs may be different, looking beyond that and sacrificing for the better good is always best in any decision. References Bio. (2016). Aristotle Biography philosopher (c.384 BCE- c.322 BCE) www.biography.com Brooks, L., Dunn, P. (2014). Business and Professional Ethics for Directors, Executives Accountants. Cengage learning. Free dictionary (2015). Philosophy. www.freedictionary.com P.I.E. (2006). Philosophy 302: Ethics Aristotle’s Ethics. Philosophy.lander.edu Toole, J. (2004).Show MoreRelatedThe Life of John F. Kennedy800 Words   |  4 PagesEdwin Schlossberg and had 3 children John, Rose , and Tatiana. John is currently 21 and attending Yale University. Rose is deceased along with her sister Tatiana.After the death of John F Kennedy , Jackie soon married Aristole Onassis in 1968 and was once again a widow when Aristole died of old age he was 69. Jackie died short after in May 19, 1994 of a form of cancer called Lymphatic system she was 64 when she died. â€Å" John Kennedy was the youngest president in office he was elected president onRead MoreContributions of Famous Scientists to Chemistry Essay example856 Words   |  4 Pagesgeneral chronological categories. These four categories are prehistoric, beginning of the christian era, end of 17th century(alchemy) traditional chemistry and modern chemistry. Some of the very first recorded chemist where men like democritus and aristole. These men where alive in the B.C era and have a lot of influence on how we do modern chemistry today. Starting with Democritus the man who claimed that the atom is the simplest unit of matter. Democritus of ancient greece asked the question couldRead MoreVirtue Ethics And Teleology1256 Words   |  6 Pagesits relevance to the culture of the given subject. Virtue is the philosophy developed by Aristole and other Ancient Greeks. It’s the quest to understand and live a life or moral character. This character based approach to morality assumes that we acquire virtue through practice. By practicing being honest, brave, just, generous, and etc. A person develops a honorable and moral character. According to Aristole by honoring virtuous habits, people will likely make the right choice when faced with ethicalRead MoreDiscuss The Importance Of Oracles To Oedipus Rex1620 Words   |  7 Pagesto definition by Aristole, there are only five things that can describe a tragedy. Tragedy is an imitation of an action that is serious, complete, and possessing. Each being used in the different parts in the mode of action and affecting through pitty and fear. Oedipus begins his journey as a very successful king. However, as the play continued he realizes that he has married his mother and has daughters with her who also happen to be his sisters. With this being said, Aristole believed that aRead MoreSimilarities Of Plato And Aristotle1617 Words   |  7 Pagesbe a negative and possessive feeling that doesn’t bring any positive the audience infers this when Plato said â€Å"Love is a serious mental disease,† Plato interprets it as a mental disease because most of the time love brings negative things with it. Aristole also mentioned several times love and it’s effect he said, â€Å"love is composed of one soul inhabiting in two bodies† this shows how Aristotle had a whole different opinion on love then Plato did Aristotle believed that love was actually a positiveRead MoreEssay about Justice901 Words   |  4 Pagesmakes equality. When we wish for quot;personal equalityquot; with people, we wish to deny what we really are and allow for superficialness. We become so obsessed with our possessions that we forget who we are and the beauty of our differences. Aristole said that quot;it is unjust to treat unequal things equallyquot;. All people are different, that is exactly what makes us human, so when we treat people entirely the same, we deny their identity. For example that does not mean that I should notRead MoreDemocracy in 17th and 18th Century Europe843 Words   |  4 Pages~Democracy  in  17th  Ã‚  18th  Century  Europe~      Olivia ­Opening  statement:  Ã‚     Democracy  is  essentially  a  type  of  government  that  splits  ruling  power  among  multiple   people.  This  prohibits  the  idea  of  absolute  rule  to  occur.  Throughout  history,  many  people   such  as  John  Locke,  Aristole,  and  Montesquieu  have  influenced  the  growth  of  a  government   that  doesnt  allow  absolute  control.  In  several  instances  throughout  time,  it  can  be  seen  that   absolutism  was  in  fact  not  the  best  option  because  it  resulted  in  an  inevitable  failure  in  a  Read MorePlato s Life And Accomplishments874 Words   |  4 PagesPlato, a Greek philosopher, was born in Athens, in 428 B.C. under the name Aristoles. In his youth, he was a wrestler, that’s how he got the name ‘Plato’, that was his ring name. Plato means broad or flat, his shoulder were broad and his forehead was flat. He won a few trophies for wrestling but never made it to the Olympics at Olympia. He later change more toward the arts, and he wrote plays, and poetry, but in never won in any of his writing competitions. â€Å"Having failed to win an Olympic goldRead MoreThe Path Of Happiness. . â€Å"Folks Are Usually About As Happy1359 Words   |  6 Pagesbecause their â€Å"happiness, is to lead a life that enables us to use and develop our reason, and that is in accordance with reason. Unlike amusement or plea sure, which can also be enjoyed by animals, happiness is not a state but an activity†, exclaimed Aristole. Happiness comes from doing things more outside than an inner feeling. As a person grows older, happiness starts to appear as a form of relief more than an idea of purpose. In the article, How Should Psychology Define Happiness?† by Doctor SusanRead MoreA Good Sheep Are Law Obeying Citizens Essay920 Words   |  4 PagesAlexandar became the most greatest military minds in history. In 335 B.C Aristotle returned to Athens and created the school Lyceum. The school was attached to the temple of Apollo Lyceus. Aristotle died in 322 BC a year after Alexandar the Great. Aristole was brought up on chargers for impierty, which lead him to flee to Chalais. He died at the age of sixty-two from a stomach ailment. Aristotle considers and rejects the likely candidates of riches, honor and health, but resolves that happiness is

Wednesday, May 13, 2020

Human Genetic Engineering And Eugenics - 1582 Words

Human genetic engineering and eugenics have been a largely controversial topic over the past decades. Eugenics can be popularly defined as the science of improving and enhancing a human population or person through manipulating the human genes, selective breeding, and sterilization. The end goal and desired result of eugenics is to basically create a human race or people with more desirable biological, physical, or psychological traits. Eugenics and genetic modification is a current, pressing subject; in April 2015, a group of Chinese researchers, used a new gene-editing technology, called CRISPR to â€Å"[tinker] with the genomes of human embryos† (Adams). Presently, according to CQ Researcher, â€Å"New genetic technologies allow scientists to delete a mutant gene and insert a healthy one, which†¦has the potential to eliminate inherited diseases, such as cystic fibrosis.† However, these techniques have only been used on embryos belonging to laboratory animals. Th e big question here is whether or not science and technology are crossing an ethical boundary by using these techniques and performing genetic modification on human embryos. Do humans have the right to â€Å"play God† and alter nature? Following the background and history of genetic engineering and eugenics, it can definitely be seen why ethicists and scientists are concerned. The concept and idea of passing on heritable traits was introduced by English naturalist Charles Darwin in his work The Origins of Species in 1859. Soon toShow MoreRelatedSale - Pro Plans Are 20% Off Today View Plans. Gradeproof1164 Words   |  5 PagesSentence Count: 28 Readability: 11.52 % Grade Level: 16.59 years Reading Time: 2 minutes Speaking Time: 4 minutes Type your title here... Genetic Engineering is a common theme of Gattaca, Splice, and Blade Runner. Gattaca takes place in a future where the best opportunities are given people that have the best genes, as a result from Genetic Engineering (cite). The main character Vincent has not been genetically engineered and has many health issues like Myopia (cite). He aspires to become anRead More The High Cost of Genetic Engineering Essay1207 Words   |  5 PagesCost of Genetic Engineering    Genetic research on human embryos, in correlation with the human genome, is the key to gene therapy, genetic diagnosis, and even to genetically engineered human beings.   Knowing which gene controls what trait and causes what genetic disease will arm doctors with a powerful tool to treat their patients at the molecular level.   On the other hand, this allows people to possibly manipulate genes to enhance specific traits or create the perfect baby.   Genetic researchRead MoreEugenics -Not the Way of the Future Essay721 Words   |  3 PagesEugenics in the world today has become an issue because of its many positive uses furthermore its possible consequences. It is believed by many that eugenics does more harm than good, on the other hand there are exceptions; it is not the way of the future. There is no doubt that it could be extremely useful for preventing diseases such as cancer and others before we are even born. But, with this also comes the ability to give children genes before their born that will give them talent to run fasterRead MoreGenetic Engineering in Humans Essay873 Words   |  4 Pagespre-birth enhancement. Iniquitous practices such as genetic engineering could lead to a degraded feeling in a child and conceivably end in a dystopian society, almost like the society Adolf Hitler had in mind. In the minds of some scientists genetic engineering for pre-birth enhancement is a potential for disaster. Disputes surrounding the definition of humanity, a threat to freedom of action, and eugenics question the moral ethics of genetic engineering, yet there is still belief that pre-birth enhancementRead MoreArguments Against Genetic Engineering673 Words   |  3 PagesGenetic Engineering I would consider one of the strongest arguments against genetic engineering was the statement given by the European Parliament in Cass R. Sunstein’s argument, The Constitution and the Clone. The following is an excerpt from the text book given by the European Parliament â€Å"The cloning of human beings†¦ cannot under any circumstance be justified or tolerated by any society, because it is serious violation of fundamental human rights and is contrary to the principleRead MoreThe Eugenics Movement By Sir Francis Galton1680 Words   |  7 Pageshad existed in the form of the eugenics movement since Sir Francis Galton published his book Hereditary Genius in 1869. Galton’s research suggested that intelligence is hereditary, and he believed that success is due to superior qualities that were passed down generation through generation. His research further suggested that personality traits and mental abilities were also hereditary (Plucker, 2013). Eugenic s, a term coined by Galton in 1883, advocated for human breeding restrictions so as toRead MoreEugenics Advantages And Disadvantages1452 Words   |  6 PagesEugenics is a movement aimed at improving the genetic composition of the human race. The word means well-born or good, well and the term was first coined by Francis Galton in 1883. Eugenics when done through selective breeding is typically broken down into two branches. Positive Eugenics is one branch aimed at improving the human race by encouraging the reproduction of people or populations with desirable traits. Negative Eugenics is the second branch focused on discouraging the reproductionRead MoreEugenics Essay1545 Words   |  7 PagesEugenics, the word that got its bad reputation years ago through an event that changed history: the Holocaust. First dubbed by Francis Galton in the 1880’s, the word Eugenics stemmed from the words â€Å"good† and â€Å"generation.† (Eugenics-Meanings) Eugenics mea ns the study of or belief in the possibility of improving the qualities of the human species or a human population. This improvement is done through discouraging reproduction by persons having genetic defects or presumed to have inheritable undesirableRead MoreThe Controversy Of Eugenics And Genetic Engineering1632 Words   |  7 Pagesscience of genetics. In that time, there have been some subjects that have been highly controversial: eugenics and genetic engineering. The history of these topics is colored with ethical and moral quandaries that have been topics for fierce debates since the early 1900s, and they continue to this day. Eugenics was defined by Francis Galton as â€Å"the science which deals with all influences that improve the inborn qualities of a race† (Galton 1). To put it in different terms, eugenics is about controllingRead MoreThe Importance of the Wise Use of Genetic Engineering1025 Words   |  4 PagesHuman species has been suffering at the hands of disease since their genesis. Our relation with each other is such that if one suffers in pain, his suffering and pain touches rest of the humanity. It is now in our domain to use this new technology called Genetic Engineering to ameliorate human suffering and pain. We should make sure that this technology is only used for the betterment of human species and not selectively. It is indeed horrifying to know that someone wants to use this as a tool

Wednesday, May 6, 2020

Understanding A Child Needs And Child Development - 851 Words

Understanding a child needs and the way to interact with them a key knowledge to create different ways for the child to development at their appropriate age level. In the beginning of the semester, I only have some knowledge of child development. I didn’t know the method of teaching a child are even to speak to them. After this class and participating in programs relating to children s development, such as Jumpstart, i slowly understand the importance of ways to express and take to the children. The environment that a child grows in affects a child in a certain degree that would either be negative or positive. For example, if a child is growing in a place that lack of space and material to manipulate, then the child will have lack of that skill he or she is supposed to acquire at a certain age and that will maintain for a lifetime. On the other hand, if the child have the spacious environment to implement play meaning that child would have enough material to develop the devel opmental skills that is appropriate for his or her age. In a classroom, it is best that there is enough materials for each individual that are able to use to reach the developmental skills. I think the most important thing that a classroom should have is natural lighting, which meaning that there should be windows. â€Å"In early childhood centers located in spaces not originally designed for children†¦.create a safe, healthy, and engaging environment†( Smith, 2013 p.9). I know that there are preschool thatShow MoreRelatedWhy Is It Important to Learn Child Development.?1048 Words   |  5 Pageswhy is it The Importance of why early years students should study child development.y. To provide effective learning and growing experiences for children and babies it is vital to have the understanding and knowledge of how children develop through-out their childhood years. `Early childhood studies 2007 Mary Wild p34` quotes that â€Å" without some theoretical understanding the danger is that, at best, we do things in particular ways out of professional habit and thereby run the risk of not providingRead MoreAttachment Theory on Socio-Emtionals Development of Children1435 Words   |  6 Pagesits introduction, the concept has developed to become one of the most significant theoretical schemes for understanding the socio-emotional development of children at an early stage. In addition, the theory is also developing into one of the most prominent models that guide parent-child relationships. Some of the key areas in these relationships that are guided by attachment theory include child welfare, parenting programs, daycare, head start programs, schools, and hospita ls. Furthermore, attachmentRead MoreLegal Status of Early Year Framework for Child Development1038 Words   |  4 Pagesframework for child development with respect to their needs and some of them has made it mandatory in their countries. For example, early year framework is part of the council of Australian Governments reform agenda and is a major part of the Australian Governments National Quality Framework for early childhood education and care. Similarly, some parts of America have its own early year framework for the child development, as Victoria has its own early year framework for child development. World healthRead MoreEymp 2 - Promoting Learning and Development in the Early Years995 Words   |  4 PagesEYMP 2 - Promoting learning and development in the early years For a child to develop and learn there are seven areas of learning and development located in the EYFS which are divided into two groups, prime and specific areas of learning. Personal and emotional development, physical development and communication and language. The prime areas are important as they lay the foundations for a child’s success in the specific areas of learning. He specific areas are learning are LiteracyRead MoreEssay Sociocultural Theory: Lev Vygotsky1112 Words   |  5 PagesLev Vygotsky was a Russian psychologist, born November 17, 1896, who had a wide range of interests that included the cognitive and language development of children. Vygotsky’s theories are somewhat incomplete due to his death at the young age of 38 from tuberculosis. Vygotsky faced many struggles in life that he was able to overcome, such as being a young Jewish boy who grew up in a time where the Russian District limited the number of Jews who were allowed to be educat ed at a University levelRead MoreHow Stressful Life Experiences Affect A Child s Emotional Development Essay1655 Words   |  7 Pagesemotional development, what role the schools play in a child’s emotional development and how there is a need for the Department of Education to commission a sector body to develop a framework of core content for initial teacher training (ITT), which should include child and adolescent development. A child’s emotional development is a complex, multifaceted process and one which many interaction partners play a role. To teach effectively, trainee teachers need to understand child development expectationsRead MoreDevelopment Delays in Learning1519 Words   |  7 PagesChapters 4, 5, 6, and 7 and the required readings for week 5, I will write a summarization of the impact that developmental delays and the environment have on the learning needs of children. My 6-8 page paper will include three examples demonstrating the impact of developmental delays and the environment on the learning needs of children discussed in the week 2 written assignments. I will also discuss why is it important for educators to understand typical behaviors when planning curriculum and IRead MoreThe Principles, Sequences And Stages Of A Child s Growth And Development1284 Words   |  6 Pagesstages of a child s growth and development. This includes Social development, Physical Development, Intelle ctual development and communication development. Physical development outlines the basic body changes which you begin to develop such as fine motor skills, which is like writing and so on. This also includes social development, where you will meet new people everyday and interact with them, and become more involved in the relationships you will share with others. Emotional development where youRead MorePlanning in Early Childhood1400 Words   |  6 Pageseach child, and their characteristics. Along with assessment and evaluation, educators are able to understand each child’s development, and make decisions about appropriate activities and experience to offer each child, to help foster their individual development. (Veale, A. and Piscitelli, B. 1988) This essay will discuss the Value of the Observation Process in Planning for early childhood settings, and the role of each teacher in facilitating children’s individual learning and development. Observation

Drowned Out Free Essays

Ivana Aleksich November 1, 2012 Film Review: â€Å"Drowned Out† The way this documentary was filmed shows the people who contributed to the film to living and working alongside the villagers and I found this helped capture the candid interviews and the honest and realistic impact of the way villagers of living in India due to the government dam project. Not only do people have to make a choice of whether they Move to the slums in the city, accept a place at a resettlement site or stay at home and drown. This place is not just a piece of land where they live but it is their home, their identity. We will write a custom essay sample on Drowned Out or any similar topic only for you Order Now This reminds me personally of my connection with Serbia. My family has to leave their homeland because of war and invasion. My family did not see this as moving to a better place, but saw it as their homeland being destroyed. Who they are as people dead in a place that no longer exists like it once had. This is how I see the people in the villagers, they are being forced to say goodbye to a certain part of them, their family, and ancestors forever. In the student presentation relating to the world bank and dams refers to something very important to villagers. Non-material things are what are important to the villagers. After the dam is built, I will drown out the cultural traditions of the villagers, create development of affected communities in isolation, and cultural shift, new lifestyles and attitudes. The documentary follows the villagers of Jalsindhi. This village is in Madhya Pradesh on the banks of the Narmada River about ten miles upstream from the Sardar Sarovar project. The 76 villages struggle through a battle against the dam. The lead character is Luharia Sonkaria, who is the village’s medicine man, a role that was his father’s and grandfather’s before him. The government provides them no viable alternatives. The government offers unusable land a hundred miles away or a small sum of money in compensation for their river-side land. The film documents hunger strikes, rallies, and a six year Supreme Court case, and finally follows the villagers as the dam fills and the river starts to rise. The documentary features Arundhati Roy, who has been an outspoken activist bringing international attention to the controversy. Government aims to provide electricity, irrigation and drinking water to tens of millions of people. Government is confident in this claim. The government has a completely difference views of the impact of the dam. They believe the dam will help the people and make them happy. But the government has nothing to lose from this dam project because the people in government do not live in the area that the dam is supposed to floor. The villagers have everything to lose and millions of them did. This brings up the struggle between the rich and the poor/the powerful and the less powerful. The big question that is stated in this documentary is, â€Å"For whom is this development for? † The government tries to convince the villagers that this is for them and their benefit, but in reality the villagers are not being taken into account for this dam project. This dam development is solely for the purpose the one thing that is universally valued: money and the power of government. Villagers and poor people are not at level in society where they can rely on money and power. They believe in things that the government and money could never find important. Villagers value their traditions and their old lifestyle. The government and the world bank is working to become more modern and to gain more money. For this reason alone, the government is aware of what will happen to the villagers but the government could not understand how hard this project will hurt the villagers and their lives. How to cite Drowned Out, Papers

Management of Organisations Concept of Work Motivation

Question: Discuss about the case study Management of Organisations for Concept of Work Motivation. Answer: Introduction The report will discuss in detail about the concept of work motivation. Motivation at work is something that develops from within an individual at the same time there are things that is completely beyond the individual form (Pinder, 2014). In order to understand, what motivates one at work is a study in itself and a lot of psychology is involved in this. What motivates me at work? I am motivated by many things at work and the primary thing which motivates me is doing the meaningful and concrete work for the organisation and recognising for my work (Danish and Usman, 2010). Both these elements are main factors for making me successful at work which become important motivators (Pinder, 2014). Another important motivator at work is the pay and constant appraisal in the pay. Singapore Airlines also comes with lots of perks like good lifestyle and travelling for free travelling all over the world and at the same time getting paid for traveling (Danish and Usman, 2010). The company is the five star airlines in the world. The company gave us the opportunity to develop a leading edge for the company by constantly improving and evolving in the organisation (Pinder, 2014). Factors that demotivates at work There are number of factors as well that demotivate an employee in Airlines industry. Following are the factors that demotivate me: Travelling to number of countries can be a strenuous job and this is why, the GMT difference leads to major jetlag and there is no time to recover it as we constantly moves into new cities (Chen et al, 2011). Constant travelling keeps us away from home for a very long period of time. This is why, I constantly miss home. Travelling and constant jetlag also create a lot health issues like sleeping disorder (Kultalahti and Liisa Viitala, 2014). Being on flight all the means that one has to be very careful and alert with the customer. This makes my job very demanding and I am in constant pressure to perform (Chen et al, 2011). Lack of vision for career is another factor that can lead to demotivation. Money is one part for motivation and there are number of people including me who are also looking for clear career goals and it can be a useful process to set clear, long-term objectives for employees (Chen et al, 2011). Job insecurity also impacts employees motivation. Every employee wants to be in a place where they can feel secure (Kultalahti and Liisa Viitala, 2014). Conclusion Motivation in employees and productivity can increase and at the same time improve the work environment. This increases number of factors that can affect the performance. These elements are actually quite simple to understand, easy to calculate and at the same time also add a lot of value in organisation (Kultalahti and Liisa Viitala, 2014). The report has discussed in detail about the factors that motivates and demotivates me at work. These are the some crucial and key things that can contribute in the work that works as major motivators and once people identify them; people get the power to completely revamp the work and the factors that keep them moving (Kultalahti and Liisa Viitala, 2014). Reference Pinder, C.C., 2014.Work motivation in organizational behavior. Psychology Press. Danish, R.Q. and Usman, A., 2010. Impact of reward and recognition on job satisfaction and motivation: An empirical study from Pakistan.International journal of business and management,5(2), p.159. Chen, G., Sharma, P.N., Edinger, S.K., Shapiro, D.L. and Farh, J.L., 2011. Motivating and demotivating forces in teams: cross-level influences of empowering leadership and relationship conflict.Journal of Applied Psychology,96(3), p.541. Kultalahti, S. and Liisa Viitala, R., 2014. Sufficient challenges and a weekend aheadGeneration Y describing motivation at work.Journal of Organizational Change Management,27(4), pp.569-582.

Monday, May 4, 2020

Culture is both a key concept and a contested concept in anthropology. Discuss free essay sample

Culture is both a key concept and a contested concept in anthropology. Discuss. I will be discussing how culture is used in anthropology, how it has seeped out into other fields of research and also its uses in normal everyday life. I will be looking at why this key concept has been and is still contested by some anthropologists. How it has created problems in the field and how we perceive people through the concept of culture. This done through the understanding of the definition of the word culture. Then I will show others who defend its uses and its importance to the creation and development of anthropology. That it is not a matter of the concept of culture but a matter of how it is interpreted and misused throughout the classic interpretation of the word. That it is merely a word to convey a set of abstract ideas together and to discard its use is not pragmatic as the word is unbounded. Culture is essentially how humans adapt to survive. That is the very root of the concept of culture. From there it has developed at a pace that far exceeds biological change. It is something that is learned socially through knowledge and actions and most of it is implicit and invisible (Busse, 2013). The word in everyday life has taken on to mean the way a certain group of people live. The word has also become a status holder, as now it has come to mean that someone with Culture is someone who is of higher class and lives a more refined, quality lifestyle. This is one of the reasons that will be touched upon why anthropologists wish to remove the word. When the word is taken into the realm of anthropology, it starts to get complicated. Multiple definitions are used and it is not universally agreed upon on which definition is to be used. The use in anthropology has become a democratisation of elitist using it. This has become a problem for some anthropologists who say it starts to resemble essentialism in the sense that it is creating a clear cut comparisons between cultures (Busse, 2013). Anthropology has a key concept that underlines everything about it, and that concept is culture. Anthropologists are hesitant and are not unanimous when it comes to defining culture, yet it is undoubtedly the most key focus in all anthropological study (Abu-Lughod, 1991). What it allows anthropologists to do is create the categories of self and other. Without the development of culture as a concept, othering would be done through the concept of race, which was how it was done in the past (Abu-Lughod, 1991). Using culture rather than race  gives certain advantages. It allows for differences to be seen in a multiple sense rather than a binary way. Which in effect then allows for a hierarchy and relativizing different cultures into categories. But the most important advantage of culture is how it removes the differences from the natural and innate and removes the confines of thinking in only those terms (Abu-Lughod, 1991). It has shown us that any set of traditions, rules or customs that are ingrained in a person who belongs to a certain culture can be unlearned and the individual can then learn a new set of rules that belong to another culture. Although the idea of culture is to steer away from the limited confines of race, there are still essentialistic tendencies that freezes differences that comes along with culture. In Saids book Orientalism (1978), he discusses the differences between the orient and the occident as a clear cut geographic, racial and culturally different areas. This discourse is so rigid in the way it speaks of the people of the west and the people of the East that it could be considered to be innate and has elements of essentialism (Abu-Lughod, 1991). Most discourses in the twentieth century now focus on culture not race. They are focused on the religions and languages that attribute to the difference in political aspects such as economic power and government of a country (Abu-Lughod, 1991). It is argued by L. Abu-Lughod (1991) that culture works in anthropological discourse in a way that categorises and separates, which then carries over a sense of hierarchy in their thinking. What she suggests is for anthropologists is to stray away from the use of culture and instead use the concept of ethnographies of the particular. This issue was raised when the clear use of the self/other distinction within feminist and bi/multi-cultural anthropologists was brought up. Marilyn Strathern (1985) argued that feminists and anthropologists have different methods of organising knowledge and drawing boundaries within their practises, even though the common interest is in the differences. She notes that this is especially true when it comes to the ethnographers relationship to their current research matter. The experiences of white middle-classed heterosexual women are different to those of more discriminated nature such as lesbians, African-American women and other minorities. This difference in the type of lives lead and experiences had gives the same topic a different perspective (Abu-Lughod, 1991). Which is why when assessing anthropologic  writing, it is important to note that although ethnocentric views have been attempted to be avoided, it is often present in the form of self categorisation, and needs to be take into account. Anthropologists have used culture as a tool for analysis in a very consistent and sophisticated fashion that shows how committed they are to the concept of culture. But even through the extensive use of culture, there have been concerns about how it still freezes differences in a way much alike to race. For example the concept of the native is a term used by anthropologists that immediately others non-Western people (Abu-Lughod, 1991). Many anthropologists have argued that cultural theory has not only frozen differences but has also created cohesion where there may not be or does not need to be. Clifford said that anthropological field-workers in an attempt to enable their own authority on the knowledge of the subject matter, created a coherent cultural other and the interpretation of self and other (Clifford, 1988:112). Abu-Lughod has brought up some methods of contesting culture. These methods are techniques that have been in use in anthropological circles today by those who also agree with the concept of writing against culture. (Abu-Lughod, 1991). One method is the theoretical discussion of anthropologists. This is one of the means by which anthropologists engage one another, making it a great way to contest culture. The use of the terms discourse and practise in discussions are good indicators of anthropologists steering away from culture. As long as discourse and practise are kept safe from being changed into just other terms for culture, and are used as intended, they will allow for a social analysis without the presumed coherence that the concept of culture has came with (Abu-Lughod, 1991). Another method of contesting culture is to change the topics that anthropologists address. The subject matter and their problems can be reworked so that the use of culture is not required. The questions need to be more focused on the history and connections between the community and the anthropologist. The useless feature of these techniques is how it brings out the similarities between all societies. When the discourse is much more in-depth and personal instead of a generalisation, it becomes less culture and more about people which should be the main focus in any anthropological study. Brumann contests the idea that culture should be written off, and argues that the negative connotations are not problems with the concept itself but associated with the ways that culture is used in ways that are less common than the critics of culture in anthropology assume (Brumann, 1999). There does not seem to be a difference in the way culture is defined between classic texts and modern ones. Yet there seems to be a significant difference in the way it is interpreted by classic and modern cultural anthropologists. Culture was simply a synonym to convey the word people in early ethnographies. Yet the word culture being linked to words such as the native which has lead to the harsh degree of us and othering, has led to the word also being dragged down as a tool that is limiting and encouraging essentialistic views. Brumann then suggests that it is not the word that is the problem, but the intent in which the user uses the word. A good example would be the use of the word race. Race being a concept that was completely unfounded and outdated, it has been in a way tabooed from use in scientific text, hindering physical anthropologists, who would wish to use the word in a non-racist way that best assists their descriptions of human biodiversity (Keita Kittles, 1997). Brumann find that culture, given its misuse in the past and present, is not comparable to the way race was used and does not deserve a similar treatment. He believes that if the meaning that those anthropologists who wish to be rid of the concept of culture gave it is discarded, and the word is used to its best intended meaning, in its most optimal way, then there would be no problem with the word (Brumann, 1999). Culture although being a noun, is not something that is there like a physical object. It is a set of abstract or arbitrary aggregate that come together to be identified as a culture (Brumann, 1999). Not unlike how for example, how a set of individual trees, streams and animals come to form a forest. Cultures are not bound by natural boundaries as it is a concept of many ideas coming together and so are only bounded by people (Brumann, 1999). I have discussed the importance of the concept of culture in the world we live in and the field of anthropology. There are those who believe that culture should be a concept that is hindering anthropology and we have no need for it anymore. There are those who defend its uses, saying that it is not the concept that fails, but the people who misunderstood the interpretation of culture gave the impression that culture is something to be avoided. That when it is used in the way it was meant, it is a tool that cannot be replaced by another word as it has become to  ingrained in our understanding of the world and societies. I believe that the concept itself is not the issue, but there are concerns that need addressing such as the use of self and other that leaks into ethnographies. Also with the need for having to gain knowledge of the whole culture and not smaller niches within the community. But culture as a whole is a concept that has allowed us to remove ourselves from the days of thinking in terms of race. It has allowed for societies to be more accepting of others as culture is something of value. It is a concept that I believe if used in the optimal way as Brumman puts it, can be a tool for aiding anthropology and less in hindering it. References Abu-Lughod, Lila, ed. 1991. Writing against Culture. Edited by R. G. Fox, Recapturing Anthropology: Working in the present. Santa Fe: School of American Research. Brumann, Christoph. 1999. Writing for culture: why a successful concept should not be discarded. Current Anthropology 40. Busse, Mark. 2013. Anthropology 203 Lectures. University of Auckland. Clifford, James. 1988. The Predicament of Culture: Twenthieth-century Ethnography, Literature, and Art. Boston: Harvard College. Keita, Kittles. 1997. The persistance of racial thinking and the myth of racial divergence. American Anthropologist 99:534-44. Said, Edward. 1978. Orientalism. Michigan: Pantheon Books.

Saturday, March 28, 2020

TOK- the ways of knowing Essay Example

TOK- the ways of knowing Paper The main reason why I positioned perception above the other four WOKs is because perception, among others, is the one tool that is absolutely necessary to gain knowledge. Regardless of how much knowledge of reasoning, emotion, or language you possess, it all revolves around the idea of perception. Perception, in my view, is the ability to understand various matters at ease, a skill which represents basis of the course. I believe it is perception that ultimately gives shape to ambiguous thoughts and develop them further as it allows you to more fully apprehend ideas. What is made out of perception, however, is strictly due to peoples pasts. People see things differently based on what theyve been through- how people perceive the same matter can range all the way from the extreme positive to the extreme negative depending on personal experience. Real- Life Example- The concept of the half filled/ emptied bottle is completely to do with perception. If a person has experienced hardship and struggled through it, that person will most likely say that it is half filled because he is gracious for everything he/she can get. In contrast, if a person who has enjoyed wealth throughout his/her life, he/she will more likely say that the bottle is half emptied because he/she can get whatever he/she wants without going through hardships. 2. Reason Reasoning, the ability to use logic to differentiate right or wrong, is another essential element of TOK. One of the main reasons for positioning the WOK of reasoning as the second most important is that reasoning allows people to piece together scattered ideas into one. We will write a custom essay sample on TOK- the ways of knowing specifically for you for only $16.38 $13.9/page Order now We will write a custom essay sample on TOK- the ways of knowing specifically for you FOR ONLY $16.38 $13.9/page Hire Writer We will write a custom essay sample on TOK- the ways of knowing specifically for you FOR ONLY $16.38 $13.9/page Hire Writer Reasoning uses past experiences to decipher bits of thoughts and ideas into various ideas depending on personal experiences and what people are taught. In other words, different people come up with different results through the process of reasoning. Yet, the gap in the variation in the results is not as large as perceptions. Unlike perception, the results obtained through reasoning do not extend as far as that of perception as reasoning is more rational, which is one of the main reasons perception is ranked higher than reason. Real-Life Example-. When presented numerous clues that lead to one another, you need reason to piece them together. For example, if given a math problem like x + 3 = 5, you can automatically know that x = 2 if you think logically as the answer is always rational regardless of your past experiences. 3. Emotion Emotion is a tool used for sharing thoughts. It tends to personalize transmitted information into the way people want it to be shared. This altered information then gets shared among people which influences the way the people perceive the ideas. While emotion helps people perceive matters personally, the extent to which emotion influence people has a limit that only reasoning and perception can exceed. Real- Life Example- When talking to others, you may want to share an idea that possesses a positive connotation. However, due to emotion, you may alter the tone of the way you speak and therefore, slightly changing the implication of what you say, and possibly making it negative. 4. Language Putting language as the least important quality among the four WOKs is not to show that language is unimportant or unnecessary, in fact, quite the opposite. Language is another vital part of human understanding as well as the others. Without language, communication would be nowhere near as explicit and the exchange of thoughts would be hindered. Even if there are other ways of communication, such as gesturing, no other methods are more efficient than simply communicating through the use of language. However, compared to the other WOKs, the use of language is just a tool to exchange ideas derived from perception, reason, and emotion. Without the other WOKs, no thoughts can be shaped and subsequently shared through language. Yet, without language, thoughts can still be exchanged, just in a more restrained and less lucid matter, although, still doable. The more accessible substitutes for language are the main reasons why I positioned language as the least important WOK among reason, perception, and emotion. Expressions, movements, body language, and several more, for example, can all be reduced substitutes for language. Real- Life Example- When talking to others, there are several ways of expression yourself, the most effective being the use of languages. For example, when you order for something in McDonalds, rather than pointing at the picture vaguely and unclearly, you can simply say the name of the food making it easier for the idea to be transferred. Show preview only The above preview is unformatted text This student written piece of work is one of many that can be found in our International Baccalaureate Theory of Knowledge section. Download this essay Print Save Not the one? Search for

Saturday, March 7, 2020

8 Rules About Punctuation and Quotation Marks

8 Rules About Punctuation and Quotation Marks 8 Rules About Punctuation and Quotation Marks 8 Rules About Punctuation and Quotation Marks By Mark Nichol The use of full or partial quotation marks or of paraphrases calls for attention to detail and adherence to a few punctuation rules. Notice that the examples below are deliberately incorrect. 1. â€Å"Most agree the word means something like: This stream meanders through something red.’† The words in the quotation collectively serve the grammatical function of a noun and are not set off from the attribution by a comma or a colon. 2. â€Å"The motto, ‘Might makes right’ applies here.† In this similar case, â€Å"the motto† is an appositive for â€Å"Might makes right,† just as in â€Å"the Web site DailyWritingTips.com,† â€Å"the Web site† is an appositive of the site’s name: â€Å"The motto ‘Might makes right’ applies here.† To insert a comma implies that this is the only existing motto (though there should then be one after right as well to complete the restrictive phrase). 3. â€Å"According to the book, at the first sign of an outbreak, ‘Children were whisked home from summer camps in the middle of the night.’† If an incomplete quotation is completed by a preceding paraphrase, lowercase the first word of the partial quotation unless it is a proper noun. In this case, â€Å"at the first sign of an outbreak† substitutes for the missing introductory phrase: â€Å"According to the book, at the first sign of an outbreak, ‘children were whisked home from summer camps in the middle of the night.’† In scholarly writing, the first letter of children should be bracketed to clarify that it was capitalized in the original source, but that nicety is unnecessary in general. 4. â€Å"He concluded that what America needs most is a â€Å"guiding belief† for citizens, industry, and government.† This sentence is essentially correct, but when a partial quote consists of such a brief phrase, ask yourself whether the quotation marks are justified; why not just paraphrase the entire sentence?: â€Å"He concluded that what America needs most is a guiding belief for citizens, industry, and government.† 5. â€Å"Her response was that she had ‘definitely locked the door on my way out.’† A writer might deem it crucial to retain a partial quote, but if the speaker uses the first person, the quotation won’t fit the reportorial third-person framing, and a paraphrase is necessary: â€Å"Her response was that she had definitely locked the door on her way out.† (Alternatively, you could paraphrase part of the direct quote â€Å"Her response was that she had ‘definitely locked the door’ on her way out† but, again, with diminishing returns.) 6. â€Å"The question is which selection is better?† This is a conjectural question not literally stated, so it is only tangentially related to the other examples here, but it’s important to point out that such constructions should include a comma: â€Å"The question is, which turnoff did she take?† (However, when the sentence is not stated as a question, the comma should be omitted: â€Å"The question is which selection is better.†) 7. When asked to clarify his earlier statement, he said: ‘I have nothing to add.’† Writers frequently introduce a statement with a colon rather than a comma, but this construction is awkward, because a colon invites the reader to put on the brakes, rather than just slow down, a fleeting action the more flexible comma invites: â€Å"When asked to clarify his earlier statement, he said, ‘I have nothing to add.’† (See also the second example, above.) Do retain the colon, however, when the attribution is an independent clause, as here: â€Å"He made this shocking public statement: ‘I think there is a fair chance Perth will be the twenty-first century’s first ghost metropolis.’† 8. â€Å"‘This [the subway bombing] is a minor thing that will develop into something major,’ she added.† When scholarly standards or journalistic integrity demands an exact quotation, but a full statement isn’t available, here’s the conventional but clunky solution: Provide the rest of what the speaker or writer meant to say or the definite noun they didn’t provide in order to fortify your class or reporting assignment within brackets. But note that the bracketed insertion should replace, not supplement, the indefinite subject: â€Å"‘[The subway bombing] is a minor thing that will develop into something major,’ she added.† Alternatively, especially in less-than-formal contexts (or even in newspaper reporting I won’t tell), feel free when you are certain of the intended specifics to employ a handy technique called silent correction. In other words, bail on the brackets: â€Å"The subway bombing is a minor thing that will develop into something major,† she added.† Want to improve your English in five minutes a day? Get a subscription and start receiving our writing tips and exercises daily! Keep learning! Browse the Punctuation category, check our popular posts, or choose a related post below:10 Grammar Mistakes You Should AvoidCapitalization Rules for the Names of GamesDozen: Singular or Plural?

Wednesday, February 19, 2020

Intercultural Communications Research Paper Example | Topics and Well Written Essays - 1500 words

Intercultural Communications - Research Paper Example From the day of its discovery until today, coffee has been a part of many nations’ customs and occasions. With a considerable market share, coffee, regardless of its kind, proves to be an interesting part of every culture, and that is why it is an important subject of research in the field of social studies. Tracing back the roots of coffee culture leads to rediscovering history, particularly how customs and traditions have evolved with the presence of the famous beverage. Coffee  is one of the most popular beverages in the world (Villanueva et al. 20-40). It would be difficult to find a restaurant, hotel, or any place that is not serving coffee. The popularity of coffee makes it an important part of human culture. Likewise, people cannot easily ignore its popularity due to its marketability and health benefits. A brief background of coffee, its origin and evolution is beneficial to know how it has become a part of human culture. Coffee is sourced from roasted  seeds of th e  coffee plant commonly known as coffee beans. The earliest trace of coffee dates back to the ninth century A.D. in southern Arabia (Weinberg and Bealer 75). Then from Ethiopia, it spread to Egypt and Yemen, then to Persia, Northern Africa, Turkey, and Armania, then to Italy and the rest of Europe; to Indonesia; and to the Americas (Meyers). Aside from being a part of many households’ breakfast, coffee has taken part in important historical occasions. For instance, it was employed in religious ceremonies in Africa and Yemen. Likewise, coffee has attracted much global attention as it was banned in  Ottoman Turkey in the 17th century due to political reasons (Hopkins). Moreover, it was also implicated in revolutionary political activities in Europe (Hopkins). These incidents associated with coffee make it a controversial beverage, a really important part of human history and culture. As a commodity, coffee is one of the most important agricultural products in the world. Mo re than 80 countries cultivate coffee (â€Å"Coffee Drinking Culture†). Likewise, coffee was at the top of agricultural export for 12 countries in 2004 (FAO Statistical Yearbook, 24) and was the world's seventh-largest legal agricultural export by value in 2006. To date, its production amounts to US$14 billion yearly (FAOSTAT Core Trade Data). The important role that coffee has played throughout history asserts the practice of drinking coffee as part of the universal culture. Due to its flavor, aroma, and energizing effects, coffee has become a popular beverage to complement home meals, and also serves as a socializing medium among people of different cultures. The role of coffee as a socializing medium may be referenced to the Les Deux Magots, a cafe formerly located in Rue de Buci in France. This cafe used to be the social hub of many literary writers in the 1900s. The place served as a venue for their meetings, literary sharing, and story telling. Indeed, in his book title d, â€Å"A Movable Feast† Ernest Hemingway (72), a world-acclaimed writer recalls his memories of Les Deux Magots as it used to be a rendezvous for all literary enthusiasts in the early 1900s. in Hemingway’s story, he and his friends used to go to Les Deux Magots for coffee before going home finally. Moreover, other literary authors such as Steve Matchett, Abha Dawesar, Vladimir Nabokov, and Craig Ferguson have their own reminiscences of the place in their own respective works. This shows not only the

Tuesday, February 4, 2020

Ethical Issues and Planning the Audit Essay Example | Topics and Well Written Essays - 1000 words

Ethical Issues and Planning the Audit - Essay Example The audit revealed certain flaws in the internal control. If after the auditor has undertaken alternative procedures to assure himself on the reliability of the report there is no assurance that the financial statement is reliable for use by any user. John the auditor may have discovered the poor internal control procedures of the company on inventories and he believes that the valuation method used in estimating the inventories will compromise the integrity of the report and a potential investor to take a risk beyond what he is willing to take if he knew the true values of the inventories. The use of estimates in the valuation of inventories instead of the cost or market which ever is lower rule is a violation of the generally accepted standard. The deviation from the standard must be disclosed and it is the duty of the auditing firm to show the effect on the inventory and on the overall reliability of the financial condition of the company under audit. Is the inventory material in relation to the overall assets of the firm, or of the current assets Was there a physical count made and was the auditor present during the physical count and such count was used in th computation of the inventory These are all considered in assessing whether the opinion of the auditor is unqualified, or a disclosure is necessary. Despite these once there is deviation from the standard of reporting is presenting the inventory values that materially affect the accuracy of the values reported it is the auditor's duty to report. John should submit his report and the outcome of his meeting with Chandler on the matter of substitution by Chandler to the firm's quality assurance board. The report should clearly state that John's findings and the alternative procedures he did, considering the poor internal and complaints of customers on merchandise defects, etc., have not cured the unreliability of the financial report. The Board will be the final arbiter that will decide whether indeed a disclosure is necessary in the report in the "Notes to financial statements" section, or a qualified opinion. Chandler may have ulterior motives in making an unqualified opinion and scope of work, etc. Question 2: Planning the Audit To: Audit Partner Re: Potential problem areas and their impact on the audit plan for Moss Green Ltd. We have undertaken the initial steps in our audit of the above-mentioned client through interviews of their key management officers who might likely be our sources of vital information and approval on items that will be included in the audited financial reports. The officers are Mr. Tom Green the managing director and Wendy Chong who is in charge of the financial affairs of the client particularly the management of the investments and foreign funds that are hedged to secure foreign currency fluctuations. We are concerned on possible problems in the presentation and disclosures of accounts. We learned from the previous auditor that Tom Gross does not have much interest on the financial affairs and he has delegated these to Wendy Chong. The possible problem with Wendy Chong is her refusal to allow disclosures on certain investments that has implications on valuation and contingent liabilities, particularly on hedged accounts. The concern is much more because Wendy enjoys a incentive compensation scheme which my tempt